2

I'm learning calculus, and I was given this information:

$$\lim_{x \to a} (f(x))^n = (\lim_{x \to a} f(x))^n$$ $$\lim_{x \to a}(f(x) + g(x)) = \lim_{x \to a} f(x) + \lim_{x \to a} g(x)$$ $$\lim_{x \to \infty}\frac{1}{x}=0$$ And I have to find this limit: $$\lim_{x \to \infty} \left(1 + \frac{1}{x}\right)^x$$ Why can't I just use my first rule to get: $$\left(\lim_{x \to \infty} \left(1 + \frac{1}{x}\right)\right)^x$$ then... $$\left(\lim_{x \to \infty} 1+\lim_{x \to \infty} \frac{1}{x}\right)^x$$ finally... $$(1 + 0)^x=1^x=1$$ I know the limit is actually equal to $e$ which is around $2.71828$, so why am I able to manipulate the limit to make it 1?

name
  • 395
  • 8
    In the first rule, the exponent is constant. In $\bigl(1 + \frac{1}{x}\bigr)^x$, it isn't. – Daniel Fischer Sep 07 '17 at 20:32
  • 1
    The problem I see is in the end. You are taking "partial" limits. You are first taking the limit of the base, and wait with the exponent. So the limit of the base is $1+0=1$, and AFTER that you incorporate the exponent into your limit. That, you can not do!. You have to take the limit "in one go". In that case, the base is never really $1$, and the exponent keeps climbing. I like your post though +1 – imranfat Sep 07 '17 at 20:32
  • This gets asked a fair bit. For instance, https://math.stackexchange.com/questions/1728752/intuitive-explanation-for-why-left1-frac1n-rightn-to-frac1e/ – Patrick Stevens Sep 07 '17 at 20:42
  • @PatrickStevens :-/ It's always annoying finding those dupes of questions that are almost like the highly upvoted ones, but aren't quite, and you know the right dupe exists but can't find it because most people don't bother with these questions. – Simply Beautiful Art Sep 07 '17 at 20:45
  • @SimplyBeautifulArt Sorry for asking a duplicate question but I can't find a question like it for some reason. – name Sep 07 '17 at 20:48
  • Personally, I do not think there exists a perfect dupe. The best I can recall from memory is basically along the lines of "Why doesn't $\lim(1+1/x)^x=1^\infty=1$?" or something like that, but I don't know of any that asks why the first rule is invalid here. – Simply Beautiful Art Sep 07 '17 at 20:51
  • And this: https://math.stackexchange.com/q/370125/9464 –  Sep 07 '17 at 21:23

4 Answers4

7

Unfortunately, the rule

$$\lim (f(x))^n=(\lim f(x))^n$$

Does not work against

$$\lim(f(x))^{g(x)}\ne(\lim f(x))^{g(x)}$$

Note that this makes no sense. For example, you would be claiming that

$$\lim_{x\to2}x^x\stackrel?=2^x$$

But clearly we actually have

$$\lim_{x\to2}x^x=2^2=4$$

Note that one results in a function, while the proper limit is a constant.

Assuming all of the following limits exist and we don't get an indeterminate form, we actually have

$$\lim(f(x))^{g(x)}=(\lim f(x))^{\lim g(x)}$$

However,

$$\lim_{x\to\infty}\left(1+\frac1x\right)^x=1^\infty$$

Which is an indeterminate form. A quick 'why?' can be seen from $1^x=1$, but $1.1^\infty=\infty$ and $0.9^\infty=0$.

Historically, this limit was one of the first definitions of $e$, one of Euler's constants,

$$\lim_{x\to\infty}\left(1+\frac1x\right)^x=e\approx2.71828182845904523\dots$$

2

If we look at $x=n\in \mathbb N$ a large natural number, we can use the binomial theorem to expand the bracket and get all positive terms $$\left(1+\frac 1n\right)^n=1+n\cdot \frac 1n + \text {other positive terms}\gt 2$$

Since we have sums larger than $2$ for at least some arbitrarily large $x$, the limit, if it exists, must be greater than $2$


Using the binomial theorem, and looking at the $n\cdot \frac 1n$ term, we see how the limit depends on the interaction of the $n$ inside the bracket and the $n$ which appears in the exponent. In your analysis you took the exponent outside the limit and lost this crucial interaction - look at the point at which you take two limits inside the bracket, and raise the result to the power $x$.

Mark Bennet
  • 100,194
  • 2
    This explains why the result produced by this technique is wrong, not why the technique is illicit. The OP asks for the latter. – Did Sep 07 '17 at 20:35
  • @Did I have added a note, which I think may be helpful to show in a practical way why the method adopted goes wrong. – Mark Bennet Sep 07 '17 at 20:49
  • @MarkBennet i liked your comment so +1 Because your comment add usefull informations to the question... – user577215664 Sep 07 '17 at 21:00
2

I think the confusion comes from the typical notation that we use in calculus. The limit notation $ \lim_\limits{x\rightarrow a}f(x) $ is used to denote the limit of a function at $a$ if it exists. Now notice that $x$ is just a "dummy" bound variable and changing it to $ \lim_\limits{y\rightarrow a}f(y) $ does nothing.

This notation is used purely for convinience instead of something like $\lim\limits_\infty f$, so that we can write expressions like $$\lim\limits_{x\rightarrow \infty}\left(1+\frac{1}{x}\right)^x$$ instead of the more verbose $$ \lim\limits_\infty f, \ \ \ \ \ \text{ where } f: \mathbb{R} \longrightarrow \mathbb{R}, \ \text{and for all } x \in \mathbb{R}, \ f(x) = \left(1+\frac{1}{x}\right)^x.$$

Now if you look at your first rule you will see that the $n$ in the power of $f(x)$ is free variable and is not bound to the limit notation and that's the only case when that rule works.

When you write down $$\left(\lim\limits_{x\rightarrow \infty}\left(1+\frac{1}{x}\right)\right)^x = \left(\lim\limits_{y\rightarrow \infty}\left(1+\frac{1}{y}\right)\right)^x $$ you actually mean $$ (\lim\limits_\infty g)^x, \ \ \ \ \ \text{ where } g: \mathbb{R} \longrightarrow \mathbb{R}, \ \text{and for all } y \in \mathbb{R}, \ g(y) = \left(1+\frac{1}{y}\right).$$ which is the limit of a completely different function $g$ risen to the power of some arbitrary number $x$ ($x$ is free variable here).

gcc-6.0
  • 695
0

First the rule you have used is :

$\displaystyle \left\lbrace \begin{array}{l}\displaystyle \lim_{x\to \infty}g(x)=\alpha\\\displaystyle \lim_{X\to \alpha} f(X)=\ell\end{array}\right.\implies \displaystyle \lim_{x\to +\infty} f\left(g(x)\right)=\ell$

Now let $g:n \to1+ \dfrac{1}{n}$ and $f_n:x\to x^n$

You agree $f_1 \ne f_2\ne f_3$ ($f_1:x\to x, \quad f_2:x\to x^2....$), these functions are differente.

And the rule you have used, $f$ must be the same function.

Stu
  • 1,690